LSAT and Law School Admissions Forum

Get expert LSAT preparation and law school admissions advice from PowerScore Test Preparation.

User avatar
 Dave Killoran
PowerScore Staff
  • PowerScore Staff
  • Posts: 5853
  • Joined: Mar 25, 2011
|
#87693
Complete Question Explanation
(The complete setup for this game can be found here: lsat/viewtopic.php?t=1469)

The correct answer choice is (B)

This is an unusual question. The stem removes P and W from the possible course list and then asks you to identify two other courses that can also not be taken. Essentially, this question asks for the two courses that can be added to P and W to create the pool of four courses not taken, which, given the constraint that at least three courses must be taken, means that the remaining three courses would have to be taken. With this information, the questions appears as follows:

G4-Q20.png

An analysis of the five courses that remain in consideration shows that M is the most restricted course and that L is the least restricted course. Thus, an attractive answer to this question would include M but not include L. This is because taking out the most restricted course and leaving in the least restricted course increases the chances that a viable group can be formed. For example, if an answer does not contain M, that means that M must be in the group of three courses taken, and because M conflicts with several other courses, the chances of a viable group being formed are reduced. If M is in an answer choice, then M is not in the group of three courses taken, causing fewer conflicts. With L, when L is not in an answer choice, L is in the group of the three courses taken, and because L is random, L does not conflict with any other course. On the contrary, when L is in an answer, L is not in the group of three courses taken, meaning another course that does conflict with at least one other course is in the group of three courses taken.

The only answer that meets the criterion of including M and not including L is answer choice (B). Let’s examine that answer more closely.

In (B), when H and M are not taken, the remaining three courses that must be taken are L, S, and T. Because S and T do not conflict with each other, this is a viable group. Hence, H and M can be added to P and W as the four courses not taken, and answer choice (B) is correct.

Remember, not all variables are equal! Some are more powerful, some are more restricted, and so on. In questions such as #19 and #20, you must use your evaluation of the variables to your benefit by considering what is possible when certain groups of variables interact.
You do not have the required permissions to view the files attached to this post.
 khana87
  • Posts: 11
  • Joined: Dec 24, 2011
|
#5240
I am not quite sure about number 20.

I don't see why e) or c) cant be the answers. Here's what I think:
If M---> NOT P + NOT T
T OR P -----> NOT M

IF W ----> NOT P + NOT S
P OR S ----> NOT W

So PTM cannot be in together as well as PSW. Since P and W are not included.
Couldn't the student also not take say theater and statistics as well?

I don't see why just b is the answer i guess. Thanks!
 Steve Stein
PowerScore Staff
  • PowerScore Staff
  • Posts: 1153
  • Joined: Apr 11, 2011
|
#5252
Hi,

In that game, it's important to remember that the student has to take at least three courses. Question 20 begins by ruling out both P and W. This leaves only five subjects--H, L, M S, and T--and we can only exclude a maximum of two more.

If we exclude H and M, as provided by answer choice B, we are left with L, S, and T. No issues.

But what about answer choice C? If we exclude H and S, we are left with L, M, and T. As you correctly pointed out, T and M cannot go together.

And answer choice E? Leaving out S and T, we are left with H, L and M. As you pointed out, H and M do not get along.

I hope that's helpful--let me know--thanks!

~Steve
 khana87
  • Posts: 11
  • Joined: Dec 24, 2011
|
#5320
I get it! Thank you :-D
 Basia W
  • Posts: 108
  • Joined: Jun 19, 2014
|
#16275
Good evening,

I am not sure why, but given the negative rules in this game I had a very hard time answering questions 20-22. I wasn't sure how to determine what I knew given what I didn't know. For instance, with question #20 if they do not take physics, there is a possibility that they could be taking music and writing, and if those are in P,T,H are our and P,S are out...I wasn't sure how to deal with these rules in these particular questions. For the other questions, I had difficulty determining what must be true.

thank you for your help,

Best,

Basia
 Ron Gore
PowerScore Staff
  • PowerScore Staff
  • Posts: 220
  • Joined: May 15, 2013
|
#16301
Basia,

You're right to view this game in context of the negative rules. All but one of the variables in this game (i.e., L) show you that at least one other variable must be excluded from the list of courses taken. Some of the variables (e.g., M) are more influential than others. The key to approaching this entire game is to get a fix on the variables that are influential, forcing other variables out (e.g., M), and those that are not influential (e.g., L and T).

With that focus, you can be more intuitive in terms of how to approach each of these questions. For example, in Question 20, you start off knowing that 2 out of the 7 variables (P and W) are already eliminated. You only have 5 variables remaining, and you must have at least three variables in the In group. That's a difficult proposition, since almost all of these variables show you that other variables can't be included. It would be tough to form a list of three variables that are included if M is one of the three, because M forces 2 of the remaining 5 variables out. However, L doesn't force any of the variables out. So, here your focus is on an answer choice containing M (meaning M is not taken), but not containing L (meaning L is taken).

Use that approach on each of these questions to help you increase both your speed and your accuracy when attacking this game.

Thanks, and let me know if I can help you further.

Ron
 Basia W
  • Posts: 108
  • Joined: Jun 19, 2014
|
#16316
Hello,

thank you SO much. I approached the next three questions with the same sort of format of attack-and they went much faster and smoothly. That makes a lot more sense and is a very logical way of approaching this problem- next time I begin my set up I will make sure to determine which variables are key players and which ones are less influential.

Best,

Basia

Get the most out of your LSAT Prep Plus subscription.

Analyze and track your performance with our Testing and Analytics Package.